B15 GI & Liver Questions

Réussis tes devoirs et examens dès maintenant avec Quizwiz!

47 A 19-year-old man is advised by other family members to see his physician because genetic screening has detected a disease in other family members. On physical examination, a stool sample is positive for occult blood. A colonoscopy is performed, followed by a colectomy. *Pt is dx' w/ familial adenomatous polyposis (FAP) syndrome*. Molecular analysis of this patient's normal fibroblasts is most likely to show a mutation in which of the following genes? □ (A) APC □ (B) p53 □ (C) K-RAS □ (D) HNPCC □ (E) NOD2

(A) APC This young patient's colon shows hundreds of polyps. This is most likely a case of familial adenomatous polyposis (FAP) syndrome, which results from inheritance of one mutant copy of the APC tumor-suppressor gene (a few FAP cases are associated with DNA mismatch repair genes). Every somatic cell of this patient most likely has one defective copy of the APC gene. Polyps are formed when the second copy of the APC gene is lost in many colon epithelial cells. Without treatment, colon cancers arise in 100% of these patients because of accumulation of additional mutations in one or more polyps, typically before 30 years of age. Patients with the gene for hereditary nonpolyposis colorectal carcinoma also have an inherited susceptibility to develop colon cancer, but in contrast to patients with FAP, they do not develop numerous polyps.

13 A 68-year-old woman has had substernal pain after meals for many years. For the past year, she has had increased difficulty swallowing liquids and solids. On physical examination, there are no remarkable findings. Upper gastrointestinal endoscopy shows a lower esophageal mass that nearly occludes the lumen of the esophagus. A biopsy specimen of this mass is most likely to show which of the following neoplasms? □ (A) Adenocarcinoma □ (B) Leiomyosarcoma □ (C) Squamous cell carcinoma □ (D) Non-Hodgkin lymphoma □ (E) Carcinoid tumor

(A) Adenocarcinoma Adenocarcinomas of the esophagus are typically located in the lower esophagus, where Barrett esophagus develops at the site of long-standing gastroesophageal reflux disease. Barrett esophagus is associated with a greatly increased risk of developing adenocarcinoma.

29 A 49-year-old woman sees her physician because she has had abdominal cramps and diarrhea with six stools per day for the past month. She has a *history of similar episodes of self-limited pain and diarrhea, which have occurred several times during the past 20 years*. Each episode lasts about 2 weeks and resolves without treatment. Findings on physical examination are unremarkable, but a stool sample is *positive for occult blood*. Laboratory studies show no ova or parasites in the stool. Colonoscopy shows *diffuse and uninterrupted mucosal inflammation and superficial ulceration extending from the rectum to the ascending colon*. Colonic biopsy specimens from the area show a diffuse, predominantly mononuclear infiltrate in the lamina propria. The patient is at high risk of developing which of the following complications? □ (A) Adenocarcinoma of the colon □ (B) Diverticulitis □ (C) Primary biliary cirrhosis □ (D) Fat malabsorption □ (E) Pseudomembranous colitis □ (F) Perirectal fistula formation

(A) Adenocarcinoma of the colon This patient has clinical and histologic features of ulcerative colitis. Particularly important are relapsing and remitting episodes of diarrhea containing blood and mucus and diffuse inflammation and ulceration of the rectal and colonic mucosa. One of the most dreaded complications of ulcerative colitis is the development of colonic adenocarcinoma. There is a 20-fold to 30-fold higher risk in patients who have had ulcerative colitis for 10 or more years compared with control populations.

Question 28 A 51-year-old man undergoes routine health examination by his nurse practitioner. There are no abnormal physical examination findings except for a stool sample positive for occult blood. Colonoscopy is performed and there is a 1 cm polyp on a narrow stalk located in the descending colon at 30 cm from the anal verge. The polyp is resected and on microscopic examination shows crowded, tubular, atypical colonic-type glands. The stalk of the polyp is covered with normal colonic epithelium. Which of the following is the most likely diagnosis? A Adenomatous polyp B Inflammatory fibroid polyp C Peutz-Jeghers polyp D Ulcerative colitis pseudopolyp E Hyperplastic polyp F Crohn disease

(A) Adenomatous polyp This is the typical appearance for a tubular adenoma, also called an adenomatous polyp. Small ones are likely to be benign, and they can easily be removed. This is the value of colonoscopic screening.

14 A 53-year-old man comes to the emergency department because of marked hematemesis that has continued for the past 3 hours. On physical examination, he has a temperature of 35.9°C, pulse of 112/min, respirations of 26/min, and blood pressure of 90/45 mm Hg. He has a distended abdomen with a fluid wave, and the spleen tip is palpable. What liver disease is most likely to be present in this patient? □ (A) Cirrhosis □ (B) Cholangiocarcinoma □ (C) Massive hepatic necrosis □ (D) Fatty change □ (E) HAV infection

(A) Cirrhosis The findings point to portal hypertension with bleeding esophageal varices. Cirrhosis alters hepatic blood flow to produce portal hypertension.

35 A 41-year-old man has experienced progressive fatigue, pruritus, and icterus for several months. A colectomy was performed 5 years ago for treatment of ulcerative colitis. On physical examination, he now has generalized jaundice. The abdomen is not distended; on palpation, there is no abdominal pain and no masses. Laboratory studies show a serum alkaline phosphatase level of 285 U/L. Cholangiography shows widespread obliteration of intrahepatic bile ducts and a beaded appearance in the remaining ducts. Which of the following morphologic features is most likely to be present in this patient's liver? □ (A) Concentric "onion-skin" bile duct fibrosis □ (B) Copper deposition in hepatocytes □ (C) Granulomatous bile duct destruction □ (D) Periportal PAS-positive globules □ (E) Piecemeal hepatocellular necrosis at the interface of portal tracts □ (F) Portal bridging fibrosis with nodular hepatocyte regeneration

(A) Concentric "onion-skin" bile duct fibrosis This patient has primary sclerosing cholangitis; ulcerative colitis coexists in 70% of these cases. The major targets in primary sclerosing cholangitis are intrahepatic bile ducts. They undergo a destructive cholangitis that leads eventually to periductal fibrosis and cholestatic jaundice. Eventually, cirrhosis and liver failure can occur.

54 A 27-year-old man has had intermittent cramping abdominal pain and low-volume diarrhea for several weeks. On physical examination, he is afebrile; there is mild lower abdominal tenderness but no masses, and bowel sounds are present. A stool sample is positive for occult blood. The symptoms subside within 1 week. Six months later, the abdominal pain recurs with perianal pain. On physical examination, there is now a perirectal fistula. Colonoscopy shows many areas of mucosal edema and ulceration and some areas that appear normal. Microscopic examination of a biopsy specimen from an ulcerated area shows a patchy acute and chronic inflammatory infiltrate, crypt abscesses, and several noncaseating granulomas. Which of the following underlying disease processes best explains these findings? □ (A) Crohn disease □ (B) Amebiasis □ (C) Shigellosis □ (D) Sarcoidosis □ (E) Ulcerative colitis

(A) Crohn disease The clinical and histologic features are consistent with Crohn disease, one of the idiopathic inflammatory bowel diseases. Crohn disease is marked by segmental bowel involvement and transmural inflammation that leads to strictures, adhesions, and fistula.

32 A 51-year-old man has sudden onset of massive emesis of bright red blood. On physical examination, his temperature is 36.9°C, pulse is 103/min, respirations are 19/min, and blood pressure is 85/50 mm Hg. Laboratory studies show a hematocrit of 21%. The serologic test result for HBsAg is positive. He has had no prior episodes of hematemesis. The hematemesis is most likely to be a consequence of which of the following? □ (A) Esophageal varices □ (B) Barrett's esophagus □ (C) Candida albicans infection □ (D) Reflux esophagitis □ (E) Squamous cell carcinoma □ (F) Zenker diverticulum

(A) Esophageal varices Variceal bleeding is a common complication of hepatic cirrhosis, which can be an outcome of chronic hepatitis B infection. The resultant portal hypertension leads to dilated submucosal esophageal veins that can erode and bleed profusely.

55 A 50-year-old woman has a *history of peptic ulcer disease* for which she has been treated with omeprazole. She has had nausea with vomiting for the past 2 months. Upper GI endoscopy reveals *3 circumscribed, round, smooth lesions in the gastric body from 1 to 2 cm in diameter*. Biopsies are taken and microscopically show the lesions to consist of *irregular glands that are cystically dilated and lined by flattened parietal and chief cells*. No inflammation, H. pylori, metaplasia, or dysplasia is present. What is the most likely diagnosis? □ (A) Fundic gland polyps □ (B) Gastric adenomas □ (C) Hyperplastic polyps □ (D) Hypertrophic gastropathy

(A) Fundic gland polyps There is an association of fundic gland polyps with use of proton pump inhibitors and with familial adenomatous polyposis (FAP).

24 A 61-year-old man has had ascites for the past year. After paracentesis with removal of 1 L of slightly cloudy, serosanguineous fluid, physical examination shows a firm, nodular liver. Laboratory findings are positive for serum HBsAg and anti-HBc. He has a markedly elevated serum α-fetoprotein level. Which of the following hepatic lesions is most likely to be present? □ (A) Hepatocellular carcinoma □ (B) Massive hepatocyte necrosis □ (C) Marked steatosis □ (D) Wilson disease □ (E) Autoimmune hepatitis

(A) Hepatocellular carcinoma The elevated α-fetoprotein level is most suggestive of hepatocellular carcinoma, which arises in cirrhosis. The presence of HBsAg and anti-HBc indicates chronic infection with HBV, which gave rise to cirrhosis and, ultimately, to liver cell cancer. Massive hepatocyte necrosis is unlikely late in the course of chronic hepatitis and cirrhosis.

44 A 24-year-old woman gives birth to an infant at term after an uncomplicated pregnancy. Apgar scores are 9 and 10 at 1 and 5 minutes after birth. The infant's length and weight are at the 55th percentile. There is no significant passage of meconium. Three days after birth, the infant vomits all oral feedings. On physical examination, the infant is afebrile, but the abdomen is distended and tender, and bowel sounds are reduced. An abdominal ultrasound scan shows marked colonic dilation above a narrow segment in the sigmoid region. A biopsy specimen from the narrowed region shows an absence of ganglion cells in the muscle wall and submucosa. Which of the following is most likely to produce these findings? □ (A) Hirschsprung disease □ (B) Trisomy 21 □ (C) Volvulus □ (D) Colonic atresia □ (E) Necrotizing enterocolitis □ (F) Intussusception

(A) Hirschsprung disease This patient has Hirschsprung disease. The aganglionic segment of the bowel wall produces a functional obstruction with proximal distention. Atresias are congenitally narrowed segments of bowel (usually the small intestine) that occur with other anomalies.

17 A 43-year-old woman has become increasingly tired and listless over the past 5 months. She has had menometrorrhagia for the past 3 months. On physical examination, there are no remarkable findings except for a positive result on stool guaiac testing. Laboratory studies show hemoglobin, 9.2 g/dL; hematocrit, 27.3%; and MCV, 75 µm3. Pelvic ultrasound reveals an enlarged uterus. A Pap smear shows abnormal cells of probable endometrial origin. Colonoscopy is performed, followed by partial colectomy; it shows a fungating mass that is typical of adenocarcinoma of the right colon. Which of the following molecular abnormalities has most likely led to these findings? □ (A) Mutation in a DNA mismatch-repair gene □ (B) Germline inheritance of APC gene mutation □ (C) Tyrosine kinase activation owing to c-KIT mutation □ (D) Homozygous loss of PTEN gene □ (E) Inactivation of the Rb protein by HPV-16

(A) Mutation in a DNA mismatch-repair gene The figure shows a large, fungating mass that is typical of adenocarcinoma of the right colon. Such cancers are unlikely to obstruct, but they can bleed a small amount over months to years, causing iron deficiency anemia. This relatively young woman has evidence for an additional cancer, an endometrial cancer, and this combination is most likely due to an inherited mutation in one of the DNA mismatch-repair genes, such as hMSH2 and hMLH1. Homozygous loss of these genes can give rise to right-sided colon cancer and endometrial cancer. Such a mutation is typically associated with microsatellite instability.

14 After an uncomplicated pregnancy, a 23-year-old woman, G2, P1, gave birth at term to a boy of normal weight and length. The infant initially did well, but at 6 weeks, he began feeding poorly for 1 week, and his mother noticed that much of the milk he ingested was forcefully vomited within 1 hour. On physical examination, the infant is afebrile, and there are no external anomalies. The physician palpates a midabdominal mass. Bowel sounds are active. The medical history indicates that the mother and her first child had the same illness during infancy. Which of the following conditions is most likely to explain these findings? □ (A) Pyloric stenosis □ (B) Tracheoesophageal fistula □ (C) Diaphragmatic hernia □ (D) Duodenal atresia □ (E) Annular pancreas

(A) Pyloric stenosis The infant's condition occurred several weeks after birth because of hypertrophy of pyloric smooth muscle. Pyloric stenosis manifests the genetic phenomenon of a "threshold of liability," above which the disease is manifested—more genetic risks are present. The incidence in males is 1/200 and in females is 1/1000, reflecting the fact that more risks must be present in females for the disease to occur; the threshold of liability would be exceeded more easily for males born into a family with the trait. Tracheoesophageal fistula, diaphragmatic hernia, and duodenal atresia are serious conditions that are manifested at birth and are often associated with multiple anomalies. Pyloric stenosis is an isolated condition that typically occurs without other anomalies. Annular pancreas is a rare anomaly.

52 A 52-year-old, previously healthy man sustained an extensive thermal burn injury involving 70% of the total body surface area of his skin. He was hospitalized in stable condition. Three weeks after the initial burn injury, he had melanotic stools. His blood pressure was 80/40 mm Hg, and his hematocrit was 18%. Soon after, he experienced cardiac arrest and could not be resuscitated. At autopsy, where are gastrointestinal ulcerations most likely to be found in this man? □ (A) Esophagus □ (B) Stomach □ (C) Duodenum □ (D) Ileum □ (E) Colon

(B) Stomach The patient has so-called stress ulcers, also known as Curling ulcers when they occur in patients with burn injuries. The ulcers are often small (<1 cm) and shallow, never penetrating the muscularis propria, but they can bleed profusely. Similar lesions can occur after traumatic or surgical injury to the central nervous system (Cushing ulcers).

60 A 57-year-old man goes to the emergency department because of increasing abdominal pain with distention that developed over the past 24 hours. On physical examination, there is diffuse abdominal tenderness. The abdomen is tympanitic, without a fluid wave, and bowel sounds are nearly absent. There is a well-healed, 5-cm transverse scar in the right lower quadrant of the abdomen. There is no caput medusae. A stool sample is negative for occult blood. An abdominal plain film shows dilated loops of small bowel with air-fluid levels, but there is no free air. At laparotomy, the surgeon notices a 20-cm portion of reddish black ileum that changes abruptly to pink-appearing bowel on distal and proximal margins. The patient's medical history is significant only for an appendectomy at age 25. Which of the following is most likely to have produced these findings? □ (A) Adenocarcinoma of the ileum □ (B) Adhesions □ (C) Angiodysplasia □ (D) Crohn disease □ (E) Indirect inguinal hernia □ (F) Intussusception □ (G) Volvulus

(B) Adhesions The patient has acute bowel obstruction, and the findings at surgery show bowel infarction. The most common causes in developed nations are adhesions, hernias, and metastases. Adhesions are most often the result of prior surgery, as in this case, and produce "internal" hernias, where a loop of bowel becomes trapped (incarcerated), and the blood supply is compromised. Loops of bowel that become trapped in direct or indirect inguinal hernias also can infarct. When metastases are the cause, the primary site is generally known, and the cancer stage is high.

36 A 31-year-old woman has experienced increasing malaise for the past 4 months. Physical examination yields no remarkable findings. Laboratory studies show total serum protein of 6.4 g/dL, albumin of 3.6 g/dL, total bilirubin of 1.4 mg/dL, AST of 67 U/L, ALT of 91 U/L, and alkaline phosphatase of 99 U/L. Results of serologic testing for HAV, HBV, and HCV are negative. *Test results for ANA and anti-smooth muscle (ASM) antibody are positive, and the rheumatoid factor level is elevated*. A liver biopsy is done; microscopically, the specimen shows minimal portal mononuclear cell infiltrates with minimal piecemeal necrosis and mild portal fibrosis. What is the most likely diagnosis? □ (A) α1-Antitrypsin deficiency □ (B) Autoimmune hepatitis □ (C) Chronic alcoholism □ (D) HDV infection □ (E) Isoniazid ingestion □ (F) Primary biliary cirrhosis □ (G) Wilson disease

(B) Autoimmune hepatitis Autoimmune hepatitis is a chronic liver disease of unknown cause in which antibodies to hepatocyte structural components cause progressive necrosis of hepatocytes, leading to cirrhosis and liver failure. Patients tend to improve with glucocorticoid therapy.

21 A 19-year-old mother notices that her 3-week-old neonate has increasing jaundice. The pregnancy was uncomplicated and ended in a normal term birth. On physical examination, the infant now exhibits generalized jaundice, hepatomegaly, and acholic stool. Laboratory studies show total serum bilirubin of 10.1 mg/dL, AST of 123 U/L, ALT of 140 U/L, glucose of 77 mg/dL, and creatinine of 0.4 mg/dL. A liver biopsy is done, and a biopsy specimen shows marked proliferation of bile ducts, portal tract edema and fibrosis, and extensive intrahepatic and canalicular bile stasis. The infant develops progressively worsening jaundice and dies of liver failure at 9 months of age. What is the most likely diagnosis? □ (A) α1-Antitrypsin deficiency □ (B) Biliary atresia □ (C) Choledochal cyst □ (D) Congenital toxoplasmosis □ (E) Hepatoblastoma

(B) Biliary atresia Extrahepatic biliary atresia is a rare condition in which some or all of the bile ducts are destroyed. If the disease spares a large enough bile duct to anastomose around the obstruction, the problem may be correctable. In many cases, such as this one, obstruction of bile ducts occurs above the porta hepatis, however, and the only option for treatment is liver transplantation.

44 After experiencing malaise and increasing icterus for 6 weeks, a 42-year-old man sees his physician. Physical examination shows jaundice, but there are no other significant findings. Serologic test results are negative for IgM antiHAV and anti-HCV, and positive for HBsAg and IgM anti-HBc. Which of the following statements is most likely to apply to this patient's illness? □ (A) The source of the infection is a blood donation made 1 month ago □ (B) Complete recovery without sequelae is most probable □ (C) There is significant risk of development of fulminant hepatitis □ (D) There is significant risk of development of hepatocellular carcinoma □ (E) All serologic test results will be negative in 1 year

(B) Complete recovery without sequelae is most probable The patient has serologic markers for HBV (HBsAg positive), and the detection of IgM anti-HBc indicates acute infection. Most cases of HBV infection do not progress to chronic hepatitis, but a few cases are complicated by fulminant hepatitis or by progression to cirrhosis. In some patients, cirrhosis progresses to hepatocellular carcinoma, but given that most patients with HBV recover, the risk of liver cancer in a specific individual is very small.

58 A clinical study of adult patients with chronic bloody diarrhea is performed. One group of these patients is found to have a statistically increased likelihood for the following: *antibodies to Saccharomyces cerevisiae* but not anti-neutrophil cytoplasmic autoantibodies, NOD2 gene polymorphisms, 50% concordance rate in monozygous twins, higher rate of cigarette smoking, TH1 immune cell activation, *vitamin K deficiency, pernicious anemia, and gallstones*. Which of the following diseases is this group of patients most likely to have? □ (A) Angiodysplasia □ (B) Crohn disease □ (C) Diverticulitis □ (D) Ischemic enteritis □ (E) Ulcerative colitis

(B) Crohn disease These are findings of idiopathic inflammatory bowel disease most likely to be Crohn disease. The ileal involvement accounts for the vitamin K and vitamin B12 deficiencies as well as disrupted enterohepatic circulation of bile salts predisposing to gallstone formation. The inflammatory response in Crohn disease may result from inappropriate innate immune responses to gut flora, as discussed in the text.

11 Seven years ago, a 30-year-old man saw a physician because he had experienced malaise, fever, and jaundice for 2 weeks. On physical examination, there were needle tracks in the left antecubital fossa and a murmur of mitral regurgitation. Serologic test results were *positive for HBsAg, HBV DNA, and IgG anti-HBc*. He did not return for follow-up. Two years later, he was seen in the emergency department because of hematemesis and ascites. Serologic test results were similar to those reported earlier. Sclerotherapy to treat esophageal varices was performed, and he was discharged. He again failed to return for follow-up. Five years after this episode, he now sees a physician because of a 5-kg weight loss, worsening abdominal pain, and rapid enlargement of the abdomen over the past month. Physical examination shows an increased liver span. Which of the following laboratory tests is most likely to be diagnostic of this last phase of his disease? □ (A) Prolonged prothrombin time □ (B) Elevated serum α-fetoprotein level □ (C) Elevated serum ALT level □ (D) Elevated serum alkaline phosphatase level □ (E) Elevated serum ferritin level □ (F) Increased blood ammonia level

(B) Elevated serum α-fetoprotein level This intravenous drug user developed chronic HBV infection, as evidenced by the persistence of HBsAg, HBV DNA, and IgG anti-HBc antibodies. Of individuals with a history of intravenous drug use, 80% to 90% are found to have serologic evidence of HBV or HCV infection. Ruptured varices and ascites suggest that this patient subsequently developed cirrhosis and portal hypertension. His final presentation of weight loss and rapid enlargement of the abdomen suggests that a hepatocellular carcinoma has developed. In most cases, hepatocellular carcinoma can be confirmed by an elevated α-fetoprotein level.

50 A 41-year-old, previously healthy woman has noted abdominal discomfort for the past month. On physical examination, she is afebrile. Her stool does not have occult blood. Laboratory studies show normal serum total protein, albumin, AST, ALT, and bilirubin, but her alkaline phosphatase is elevated. Serologic testing for hepatitis A, B, and C viruses is negative. Abdominal CT scan shows a 9-cm right hepatic lobe mass with irregular borders. The lesion is resected and grossly has a central stellate scar with radiating fibrous septa that merge into surrounding hepatic parenchyma. On microscopic examination, the mass has prominent arteries in dense connective tissue along with lymphocytic infiltrates and bile duct proliferation. What is the most likely diagnosis? □ (A) Cholangiocarcinoma □ (B) Focal nodular hyperplasia □ (C) Hepatic adenoma □ (D) Hepatocellular carcinoma □ (E) Macronodular cirrhosis □ (F) Metastatic adenocarcinoma

(B) Focal nodular hyperplasia Focal nodular hyperplasia is a well-demarcated but unencapsulated benign lesion that is characterized by a central scar. Cholangiocarcinomas have extensive collagen deposition, but are malignant and often associated with risk factors including viral hepatitis. Hepatic adenomas can be seen with oral contraceptive use. Hepatocellular carcinomas occur after liver injury from such conditions as viral hepatitis or alcohol abuse, and they often can be multifocal. Cirrhosis involves the entire liver, not just a portion. Metastases can be solitary or multiple and can have central necrosis when large. BP7 625BP8 663PBD7 922PBD8 876

32 A 35-year-old woman visits her physician because she has noticed an increasing yellowish hue to her skin for the past week. On physical examination, there is no abdominal pain or tenderness, and the liver span is normal. Laboratory findings include hemoglobin, 11.7 g/dL; hematocrit, 35.2%; MCV, 98 µm3; platelet count, 207,600/mm3; WBC count, 6360/mm3; albumin, 3.5 g/dL; total protein, 5.5 g/dL; total bilirubin, 8.7 mg/dL; direct bilirubin, 0.6 mg/dL; AST, 39 U/L; ALT, 24 U/L; and alkaline phosphatase, 35 U/L. What is the most likely diagnosis? □ (A) Cholelithiasis □ (B) Hemolytic anemia □ (C) HAV infection □ (D) Micronodular cirrhosis □ (E) Oral contraceptive use

(B) Hemolytic anemia This patient has an unconjugated hyperbilirubinemia, which can result from hemolysis. With increased RBC destruction, there is more bilirubin than can be conjugated by the hepatocytes.

33 A 16-year-old boy who is receiving chemotherapy for *acute lymphoblastic leukemia* sees the physician because he has had pain for 1 week when he swallows food. Physical examination shows no abnormal findings. Upper gastrointestinal endoscopy shows 0.5- to 0.8-cm *mucosal ulcers in the region of the mid to lower esophagus*. The shallow ulcers are *round and sharply demarcated, and have an erythematous base*. Which of the following is most likely to produce these findings? □ (A) Aphthous ulcerations □ (B) Herpes simplex esophagitis □ (C) Gastroesophageal reflux disease □ (D) Candida esophagitis □ (E) Mallory-Weiss syndrome

(B) Herpes simplex esophagitis The "punched-out" ulcers described result from rupture of the herpetic vesicles. Herpesvirus and Candida infections typically occur in immunocompromised patients, and both can involve the esophagus. Candidiasis has the gross appearance of tan-to-yellow plaques.

56 A 65-year-old woman goes to her physician for a routine health maintenance examination. A stool sample is positive for occult blood. CT scan of the abdomen shows numerous air-filled, 1-cm outpouchings of the sigmoid and descending colon. Which of the following complications is most likely to develop in this patient? □ (A) Adenocarcinoma □ (B) Pericolic abscess □ (C) Bowel obstruction □ (D) Malabsorption □ (E) Toxic megacolon

(B) Pericolic abscess This patient has colonic diverticulosis, which may be accompanied by intermittent minimal bleeding and, rarely, by severe bleeding. One or more diverticula may become inflamed (diverticulitis) or, less commonly, may perforate to produce an abscess, peritonitis, or both.

46 A 45-year-old woman has had increasing pruritus and icterus for several months. On physical examination, she has generalized jaundice. Laboratory studies show total serum protein, 6.3 g/dL; albumin, 2.7 g/dL; total bilirubin, 5.7 mg/dL; direct bilirubin, 4.6 mg/dL; AST, 77 U/L; ALT, 81 U/L; and alkaline phosphatase, 221 U/L. A liver biopsy specimen shows destruction of portal tracts, loss of bile ducts, and lymphocytic infiltrates. Which of the following additional laboratory findings is most likely to be reported? □ (A) Positive anti-HCV □ (B) Positive antimitochondrial antibody □ (C) Elevated sweat chloride level □ (D) Increased serum ferritin level □ (E) Decreased α1-antitrypsin level

(B) Positive antimitochondrial antibody This patient has primary biliary cirrhosis, an uncommon autoimmune disorder that causes progressive intrahepatic bile duct destruction. Pruritus, conjugated hyperbilirubinemia, and increased alkaline phosphatase levels are indicative of obstructive jaundice resulting from bile duct destruction. About 90% or more of patients with this disease have antimitochondrial antibodies in the serum

39 A 48-year-old man sees his physician because he has had nausea and colicky right upper quadrant pain for the past 2 days. On physical examination, his temperature is 38.8°C. Laboratory studies show a WBC count of 11,200/mm3 with 71% segmented neutrophils, 9% bands, 13% lymphocytes, and 7% monocytes. What is the most likely diagnosis? □ (A) Acute HAV infection □ (B) Extrahepatic biliary atresia □ (C) Acute cholecystitis □ (D) Primary sclerosing cholangitis □ (E) Adenocarcinoma of the gallbladder

(C) Acute cholecystitis The symptoms are typical of acute calculous cholecystitis.

20 A 70-year-old man takes large quantities of nonsteroidal anti-inflammatory drugs because of chronic degenerative arthritis of the hips and knees. Recently, he has had epigastric pain with nausea and vomiting and an episode of hematemesis. On physical examination, there are no remarkable findings. A gastric biopsy specimen is most likely to show which of the following lesions? □ (A) Epithelial dysplasia □ (B) Hyperplastic polyp □ (C) Acute gastritis □ (D) Adenocarcinoma □ (E) Helicobacter pylori infection

(C) Acute gastritis Prolonged use of nonsteroidal anti-inflammatory drugs is an important cause of acute gastritis. Excessive alcohol consumption and smoking also are possible causes. Acute gastritis tends to be diffuse and, when severe, can lead to significant hemorrhage that is difficult to control. Epithelial dysplasia may occur at the site of chronic gastritis. It is a forerunner of gastric cancer. Hyperplastic polyps of the stomach do not result from acute gastritis, but may arise in association with chronic gastritis. Acute gastritis does not increase the risk of gastric adenocarcinoma. Infection with Helicobacter pylori is not associated with acute gastritis.

26 A 59-year-old man has had increasing difficulty swallowing during the past 6 months. There are no significant findings on physical examination. Upper gastrointestinal endoscopy shows areas of erythematous mucosa above the Z-line. A biopsy specimen from the *lower esophagus shows columnar metaplasia, consistent with Barrett esophagus*. Which of the following complications is most likely to occur as a consequence of this patient's condition? □ (A) Hematemesis □ (B) Squamous cell carcinoma □ (C) Adenocarcinoma □ (D) Achalasia □ (E) Lacerations (Mallory-Weiss syndrome)

(C) Adenocarcinoma The biopsy specimen shows columnar metaplasia, typical of Barrett esophagus. Patients with a focus of Barrett esophagus larger than 2 cm have a 30-fold to 40-fold higher risk of developing adenocarcinoma than the general population. Squamous cell carcinomas occur in the esophagus, but they do not arise in association with Barrett esophagus. Their occurrence is related to smoking and alcohol consumption.

9 A 33-year-old man who lives in New York is bothered by a low-volume, mostly watery diarrhea associated with flatulence. The symptoms occur episodically, but they have been persistent for the past year. He has experienced a 5-kg weight loss. He has no fever, nausea, vomiting, or abdominal pain. On physical examination, there are no significant findings. A stool sample is negative for occult blood, ova, and parasites, and a stool culture yields no pathogens. An upper gastrointestinal endoscopy is performed. A biopsy specimen from the upper part of the small bowel shows severe diffuse blunting of villi and a chronic inflammatory infiltrate in the lamina propria. Which of the following serologic tests is most likely to be positive in this patient? □ (A) Anticentromeric antibody □ (B) Anti-DNA topoisomerase I antibody □ (C) Antigliadin antibody □ (D) Antimitochondrial antibody □ (E) Antinuclear antibody

(C) Antigliadin antibody The clinical and histologic features suggest celiac disease. Characteristic serologic findings include positive tests for anti-transglutaminase, anti-gliadin, and anti-endomysial antibodies. This chronic disease may manifest in young adulthood but may escape diagnosis. Women are affected more than men. Celiac disease results from gluten sensitivity. Exposure to the gliadin protein in wheat, oats, barley, and rye (but not rice) results in intestinal inflammation. Gliadin sensitivity causes epithelial cells to produce IL-15, which in turn leads to accumulation of activated CD8+ T cells that bear the NK cell receptor NKG2D and damage the enterocytes expressing MIC-A. A trial of a gluten-free diet is the most logical therapeutic option. Patients usually become symptom-free, and normal histologic features of the mucosa are restored. Some patients develop dermatitis herpetiformis, and a few enteropathy-associated T-cell lymphomas.

45 A 51-year-old man has a long history of chronic alcoholism and comes to the physician because of increasing malaise for the past year. He was hospitalized 1 year ago because of upper gastrointestinal hemorrhage. Physical examination shows a firm nodular liver. Laboratory findings show a serum albumin level of 2.5 g/dL and prothrombin time of 28 seconds. Which of the following additional physical examination findings is most likely to be present? □ (A) Splinter hemorrhages □ (B) Diminished deep tendon reflexes □ (C) Caput medusae □ (D) Papilledema □ (E) Distended jugular veins

(C) Caput medusae This patient has alcoholic cirrhosis with portal hypertension. Venous collateral flow can be increased in esophageal submucosal veins, producing varices, and in the abdominal wall, producing caput medusae.

30 A 47-year-old man has experienced intermittent upper abdominal pain for several weeks. Physical examination yields no remarkable findings. Laboratory findings show total serum protein of 7.3 g/dL, albumin of 5.2 g/dL, total bilirubin of 7.5 mg/dL, direct bilirubin of 6.8 mg/dL, AST of 35 U/L, ALT of 40 U/L, and alkaline phosphatase of 207 U/L. A liver biopsy is done, and microscopic examination of the specimen shows intracanalicular cholestasis in the centrilobular regions, swollen liver cells, and portal tract edema. There is no necrosis and no fibrosis. There is no increase in stainable iron. What is the most likely diagnosis? □ (A) Chronic passive congestion □ (B) HBV infection □ (C) Choledocholithiasis □ (D) Extrahepatic biliary atresia □ (E) Veno-occlusive disease

(C) Choledocholithiasis Intermittent upper abdominal pain is a nonspecific symptom that often occurs in patients with gallstones. When a stone slips into the common bile duct, intrahepatic cholestasis occurs. This explains the conjugated hyperbilirubinemia and the increased alkaline phosphatase level.

20 A 60-year-old man with a 30-year history of alcohol abuse sees his physician because of hematemesis for the past day. On examination, he has ascites, mild jaundice, and an enlarged spleen. He also has gynecomastia, spider telangiectasias of the skin, and testicular atrophy. Rectal examination indicates prominent hemorrhoids and a normal-sized prostate. Emergent upper endoscopy shows dilated, bleeding submucosal vessels in the esophagus. Sclerotherapy is performed to control the bleeding. Laboratory studies show total protein, 5.8 g/dL; albumin, 3.4 g/dL; AST, 137 U/L; ALT, 108 U/L; alkaline phosphatase, 181 U/L; total bilirubin, 5.4 mg/dL; direct bilirubin, 3 mg/dL; prothrombin time, 20 seconds; ammonia, 76 µmol/L; and hematocrit, 25%. Which of the following findings is most likely the prominent feature in his liver? □ (A) Cholangitis □ (B) Cholestasis □ (C) Cirrhosis □ (D) Hepatitis □ (E) Steatosis

(C) Cirrhosis Portal fibrosis and nodular hepatocyte regeneration are typical features of chronic alcohol abuse. Spider telangiectasias (angiomas) refer to vascular lesions in the skin characterized by a central, pulsating, dilated arteriole from which small vessels radiate. These lesions result from hyperestrogenism (which also contributes to the testicular atrophy). The failing liver is unable to metabolize estrogens normally. Spider angiomas are a manifestation of hepatic failure. Ascites, splenomegaly, hemorrhoids, and esophageal varices all are related to portal hypertension from cirrhosis and the resultant collateral venous congestion and dilation.

6 A 23-year-old primigravida gives birth at term to a boy. Ultrasound examination before delivery showed polyhydramnios. A single umbilical artery is seen at the time of birth. It is noted that the infant vomits all feedings, then develops a fever and difficulty with respirations within 2 days. A radiograph shows both lungs and the heart are of normal size, but there are pulmonary infiltrates and no stomach bubble. What is the most likely diagnosis? □ (A) Achalasia □ (B) Diaphragmatic hernia □ (C) Esophageal atresia □ (D) Hiatal hernia □ (E) Pyloric stenosis □ (F) Squamous cell carcinoma □ (G) Zenker diverticulum

(C) Esophageal atresia An esophageal atresia is often combined with a fistula between the esophagus and trachea. Gastrointestinal obstruction in utero can lead to polyhydramnios. The presence of a single umbilical artery suggests additional anomalies are present. Vomiting in an infant risks aspiration with development of pneumonia.

38 On the day of the final examination in anatomy, a 26-year-old medical student notices that her sclerae have a slight yellowish color. She has never had a major illness. On physical examination, there are no significant findings other than the mild scleral icterus. Laboratory studies show total serum protein, 7.9 g/dL; albumin, 4.8 g/dL; AST, 48 U/L; ALT, 19 U/L; alkaline phosphatase, 32 U/L; total bilirubin, 4.9 mg/dL; and direct bilirubin, 0.8 mg/dL. The scleral icterus resolves within 2 days. Which of the following conditions is most likely to produce these findings? □ (A) Choledochal cyst □ (B) Primary biliary cirrhosis □ (C) Gilbert syndrome □ (D) Acute HAV infection □ (E) Dubin-Johnson syndrome □ (F) Ingestion of acetaminophen

(C) Gilbert syndrome This patient has Gilbert syndrome, which results from decreased levels of uridine diphosphateglucuronosyltransferase (UGT). Seven percent of individuals in the general population may have decreased levels of this enzyme, and the condition is often never diagnosed. Stress may cause transient unconjugated hyperbilirubinemia to a point that scleral icterus is detectable, when the serum bilirubin reaches about 2 to 2.5 mg/dL.

2 A 41-year-old woman who works as a tattoo artist has had increasing malaise and nausea for the past 2 weeks. On physical examination, she has icterus and mild right upper quadrant tenderness. Laboratory studies show serum AST of 79 U/L, ALT of 85 U/L, total bilirubin of 3.3 mg/dL, and direct bilirubin of 2.5 mg/dL. She continues to have malaise for the next year. A liver biopsy is done, and the biopsy specimen shows *minimal hepatocyte necrosis, mild steatosis, and minimal portal bridging fibrosis*. An infection with which of the following viruses is most likely to produce these findings? □ (A) HAV □ (B) HBV □ (C) HCV □ (D) HEV □ (E) Coinfection with HBV and HDV

(C) HCV Necrosis with portal bridging suggests chronic hepatitis. Mild steatosis is seen in HCV infection. The incidence of chronic hepatitis is highest with HCV infection. More than 50% of individuals infected with this virus develop chronic hepatitis, and many cases progress to cirrhosis. This is partly because the IgG antibodies against HCV that develop after acute infection are not protective.

52 A 69-year-old woman has had worsening congestive heart failure along with arthritis resembling pseudogout for the past 6 years. On examination her skin has a slate-grey color, her heart rate is irregular, her liver span is increased, and her spleen is palpable. Laboratory studies show an elevated hemoglobin A1c and increased serum ferritin. A mutation involving which of the following genes is most likely to be present in this woman? □ (A) α-1AT-Z □ (B) ATP7B □ (C) Hepcidin □ (D) HNF1 □ (E) UGT1A1

(C) Hepcidin Although most cases of hereditary hemochromatosis result from mutations of the HFE gene, some cases may occur from mutations of genes encoding for transferrin receptors, hemojuvelin, and rarely hepcidin. However, the main regulator of iron absorption is the protein hepcidin and all the genetic causes of hereditary hemochromatosis are associated with reduced hepcidin levels. Ordinarily the liver increases hepcidin production when iron stores are adequate, preventing release of iron from intestinal enterocytes and macrophages.

4 One year after having an acute myocardial infarction, a 55-year-old man saw his physician because of severe abdominal pain and bloody diarrhea. On physical examination, the abdomen was diffusely tender, and bowel sounds were absent. Abdominal plain films showed no free air. Laboratory studies showed a normal CBC and normal levels of serum amylase, lipase, and bilirubin. His condition deteriorated, and he developed irreversible shock. At autopsy, which of the following lesions is most likely to be found? □ (A) Acute appendicitis □ (B) Acute pancreatitis □ (C) Intestinal infarction □ (D) Acute cholecystitis □ (E) Pseudomembranous colitis

(C) Intestinal infarction The patient's history of myocardial infarction suggests that he had severe coronary atherosclerosis. Atheromatous disease most likely involved the mesenteric vessels as well, giving rise to thrombosis of the blood vessels that perfuse the bowel. The symptoms and signs suggest infarction of the gut.

16 A 28-year-old man has had increasing shortness of breath for the past year. On physical examination, he is afebrile and normotensive. Breath sounds are decreased in all lung fields. His medical history indicates that he developed marked icterus as a neonate, but he has been healthy since then. Because of a family history of liver disease, a liver biopsy is performed. The figure shows the microscopic appearance of the *liver biopsy specimen stained with PAS*. This patient is most likely at a very high risk for development of which of the following conditions? □ (A) Diabetes mellitus □ (B) Congestive heart failure □ (C) Pulmonary emphysema □ (D) Ulcerative colitis □ (E) Systemic lupus erythematosus

(C) Pulmonary emphysema The PAS-positive globules in the liver seen here are characteristic of α1-antitrypsin (AAT) deficiency. Approximately 10% of individuals with the homozygous deficiency (PiZZ phenotype) of AAT deficiency develop significant liver disease, including neonatal hepatitis and progressive cirrhosis. Deficiency of AAT also allows unchecked action of elastases in the lung, which destroys the elastic tissue and causes emphysema. .

28 A 57-year-old woman has had burning epigastric pain after meals for more than 1 year. Physical examination shows no abnormal findings. Upper gastrointestinal endoscopy shows an erythematous patch in the lower esophageal mucosa. A biopsy specimen shows basal squamous epithelial hyperplasia, elongation of lamina propria papillae, and scattered intraepithelial neutrophils with some eosinophils. Which of the following is the most likely diagnosis? □ (A) Barrett esophagus □ (B) Esophageal varices □ (C) Reflux esophagitis □ (D) Scleroderma □ (E) Iron deficiency

(C) Reflux esophagitis These findings indicate an ongoing inflammatory process resulting from reflux of acid gastric contents into the lower esophagus. Gastroesophageal reflux disease (GERD) is a common problem that stems from a variety of causes, including sliding hiatal hernia, decreased tone of the lower esophageal sphincter, and delayed gastric emptying. Patients may have a history of "heartburn" after eating.

41 Over the past 3 months, a 45-year-old woman has noticed that her skin has become progressively more yellow. On physical examination, she is afebrile and has scleral icterus and generalized jaundice. Laboratory studies show total serum bilirubin of 8.9 mg/dL, direct bilirubin of 6.8 mg/dL, serum ALT of 125 U/L, and serum AST of 108 U/L. A liver biopsy specimen shows histologic features of sclerosing cholangitis. Which of the following diseases of the gastrointestinal tract is most likely to coexist with the liver disease? □ (A) Chronic pancreatitis □ (B) Diverticulosis □ (C) Ulcerative colitis □ (D) Celiac sprue □ (E) Peptic ulceration

(C) Ulcerative colitis Sclerosing cholangitis is a serious extraintestinal manifestation of idiopathic inflammatory bowel disease, most often ulcerative colitis or, less often, Crohn disease.

37 An epidemiologic study is conducted in Singapore of patients infected with HBV. These patients are followed for 10 years from the time of diagnosis. Historical data are collected to determine the mode of transmission of HBV. The patients receive periodic serologic testing for HBsAg, anti-HBs, and anti-HBc, and serum determinations of total bilirubin, AST, ALT, alkaline phosphatase, and prothrombin time. The study identifies a subset of patients who are found to be chronic carriers of HBV. The study is most likely to show an association between the carrier state and which of the following modes of transmission of HBV? □ (A) Blood transfusion □ (B) Heterosexual transmission □ (C) Vertical transmission during childbirth □ (D) Oral transmission □ (E) Needle-stick injury

(C) Vertical transmission during childbirth

1 A 30-year-old woman sees her physician because she has had diarrhea and fatigue and has noticed a 3-kg weight loss over the past 6 months. On physical examination, she is afebrile and has mild muscle wasting, but her motor strength is normal. Laboratory studies show no occult blood, ova, or parasites in the stool. A biopsy specimen from the upper jejunum is obtained, and microscopic findings are reviewed. The patient is placed on a special diet with no wheat or rye grain products. The change in diet produces dramatic improvement. Which of the following microscopic features is most likely to be seen in the biopsy specimen? □ (A) Lymphatic obstruction □ (B) Noncaseating granulomas □ (C) Villous blunting and flattening □ (D) Foamy macrophages within the lamina propria □ (E) Crypt abscesses

(C) Villous blunting and flattening This patient has malabsorption that responded to dietary treatment. She probably has celiac disease (gluten sensitivity). The histologic features of celiac disease are flattening of the mucosa, diffuse and severe atrophy of the villi, and chronic inflammation of the lamina propria. There is an increase in intraepithelial lymphocytes.

46 A 30-year-old man has a routine health maintenance examination. A stool sample is positive for occult blood. On colonoscopy, an ulcerative lesion is seen projecting into the cecum. The microscopic *appearance suggests hereditary nonpolyposis colorectal carcinoma (HNPCC)*. Which of the following molecular biologic events is thought to be most critical in the development of such lesions? □ (A) Overexpression of E-cadherin gene □ (B) Amplification of ERBB2 gene □ (C) Germline transmission of a defective RB gene □ (D) A defective DNA mismatch-repair gene □ (E) Translocation of retinoic acid receptor α gene

(D) A defective DNA mismatch-repair gene The lesion is an adenocarcinoma, showing irregular glands infiltrating the muscle layer. Such a lesion in a 30-yearold man strongly indicates a hereditary predisposition. One hereditary form of cancer is called hereditary nonpolyposis colorectal carcinoma (HNPCC) and results from defective DNA mismatch-repair genes. As a result, mutations accumulate in microsatellite repeats (microsatellite instability) that lead to loss of transforming growth factor-β (TGF-β) receptormediated control of colonic epithelial cell proliferation and loss of pro-apoptotic BAX protein enhancing survival of these transformed cells. He could have taken NSAIDS that inhibit COX-2 expressed in most colonic adenomas and carcinomas. In contrast to familial adenomatous polyposis syndrome, HNPCC does not lead to the development of hundreds of polyps in the colon. E-cadherin is required for intercellular adhesion; its levels are reduced, not increased, in carcinoma cells. Detection of ERBB2 (HER2/NEU) expression is important in breast cancers. Germline inheritance of the tumor-suppressor gene RB predisposes to retinoblastoma and osteosarcoma, not colon carcinoma. Translocation of the retinoic acid receptor α gene is characteristic of acute promyelocytic leukemia. BP7 583-584BP8 622-623PBD7 858, 862-864PBD8 821-822

15 A 53-year-old woman has had nausea, vomiting, and mid epigastric pain for 5 months. On physical examination, there are no significant findings. An upper gastrointestinal radiographic series shows gastric outlet obstruction. Upper gastrointestinal endoscopy shows an ulcerated mass that is 2 × 4 cm at the pylorus. Which of the following neoplasms is most likely to be seen in a biopsy specimen of this mass? □ (A) Non-Hodgkin lymphoma □ (B) Neuroendocrine carcinoma □ (C) Squamous cell carcinoma □ (D) Adenocarcinoma □ (E) Leiomyosarcoma

(D) Adenocarcinoma The most likely cause of a large mass lesion in the stomach is a gastric carcinoma, and this lesion is an adenocarcinoma. Malignant lymphomas and leiomyosarcomas are less common and tend to form bulky masses in the fundus. Neuroendocrine carcinomas are rare. Squamous cell carcinomas appear in the esophagus.

31 A 42-year-old woman has had generalized pruritus for several months. The pruritus is not relieved by application of topical corticosteroid-containing creams. On physical examination, there are no remarkable findings. Laboratory findings include total serum bilirubin of 1.8 mg/dL, direct bilirubin of 1.2 mg/dL, AST of 55 U/L, ALT of 58 U/L, alkaline phosphatase of 289 U/L, total protein of 6.8 g/dL, albumin of 3.4 g/dL, and total cholesterol of 344 mg/dL. Which of the following serologic test findings is most likely to be positive in this patient? □ (A) Anti-parietal cell antibody □ (B) Anticentromere antibody □ (C) Antiribonucleoprotein □ (D) Antimitochondrial antibody □ (E) Anti-double-stranded DNA antibody

(D) Antimitochondrial antibody This patient has findings characteristic of primary biliary cirrhosis, which has a peak incidence in middle-aged women. Later in the disease, jaundice may increase with progressive destruction of intrahepatic bile ducts. Positivity for antimitochondrial antibody is a characteristic finding in most cases.

21 A 44-year-old woman has had increasing difficulty swallowing liquids and solids for the past 6 months. On physical examination, her fingers have reduced mobility because of taut, nondeforming skin. A barium swallow shows marked dilation of the esophagus with "beaking" in the distal portion, where there is marked luminal narrowing. A biopsy specimen from the lower esophagus shows prominent submucosal fibrosis with little inflammation. Which of the following is most likely to produce these findings? □ (A) Portal hypertension □ (B) Iron deficiency □ (C) Barrett esophagus □ (D) CREST syndrome □ (E) Hiatal hernia

(D) CREST syndrome Esophageal dysmotility is the "E" in CREST syndrome, the limited form of systemic sclerosis (scleroderma). Although the disease is autoimmune, little inflammation is seen by the time the patient seeks clinical attention. There is increased collagen deposition in submucosa and muscularis. Fibrosis may affect any part of the gastrointestinal tract, but the esophagus is the site most often involved.

29 A 55-year-old man has developed abdominal pain and jaundice over several weeks. On physical examination, there is right upper quadrant pain, but no abdominal distention. Abdominal CT scan shows a markedly thickened gallbladder wall. A cholecystectomy is performed, and sectioning shows a slightly enlarged gallbladder containing a fungating, 4 × 7 cm firm, lobulated, tan mass. What finding is most likely associated with this mass? □ (A) Amebic dysentery □ (B) Ulcerative colitis □ (C) Clonorchis sinensis infection □ (D) Cholelithiasis □ (E) Primary sclerosing cholangitis

(D) Cholelithiasis Almost all gallbladder carcinomas are adenocarcinomas, and most are found in gallbladders that also contain gallstones.

23 In a study of the mechanisms of gallstone formation, patients with cholelithiasis confirmed by radiographic imaging (ultrasound or CT) are analyzed to determine if there are underlying conditions that contribute to this process. The study identifies a subset of patients who have decreased urobilinogen excretion in urine and increased fecal bile acids. Despite normal values for serum bilirubin, albumin, haptoglobin, and calcium, many of these patients have a macrocytic anemia. The liver and spleen in these patients appear normal on abdominal CT. What condition is most likely to be present in this subset of patients? □ (A) Adenocarcinoma of the gallbladder □ (B) Obesity □ (C) Hemolytic anemia □ (D) Crohn disease □ (E) Chronic hepatitis B

(D) Crohn disease These findings indicate decreased bile acid and urobilinogen reabsorption in the ileum, which reduces the enterohepatic circulation and favors the formation of pigment stones. Crohn disease often involves the terminal ileum; this also disturbs absorption of vitamin B12 complexed with intrinsic factor, leading to macrocytic anemia.

11 During summer "Black and White Days," a week-long local community celebration of the dairy industry (Holstein cows are black and white), a 40-year-old man has episodic abdominal bloating, flatulence, and explosive diarrhea. On physical examination, there are no remarkable findings. Laboratory studies show no increase in stool fat and no occult blood, ova, or parasites in the stool. A routine stool culture yields no pathogens. During the rest of the year, the patient does not consume milkshakes or ice cream sodas and is not symptomatic. Which of the following conditions best accounts for these findings? □ (A) Celiac disease □ (B) Autoimmune gastritis □ (C) Cholelithiasis □ (D) Disaccharidase deficiency □ (E) Cystic fibrosis

(D) Disaccharidase deficiency Disaccharidase (lactase) deficiency is an uncommon congenital condition (or a rare acquired condition) in which the lactose in milk products is not broken down into glucose and galactose, resulting in an osmotic diarrhea and gas production from gut flora. Affected individuals do not always make the connection between diet and symptoms, or they do not consume enough milk products to become symptomatic.

8 A 36-year-old woman has become increasingly icteric for 1 month. In the past 3 years, she has had several bouts of colicky, mid abdominal pain. On physical examination, she has generalized jaundice with scleral icterus. There is tenderness in the right upper quadrant, and the liver span is normal. A liver biopsy is done; microscopic examination of the specimen shows bile duct proliferation and intracanalicular bile stasis, but no inflammation or hepatocyte necrosis. Which of the following serum laboratory findings is most likely to be present in this patient? □ (A) Markedly increased antimitochondrial antibody □ (B) Positive hepatitis C antibody □ (C) Markedly elevated indirect bilirubin level □ (D) Elevated alkaline phosphatase level □ (E) Increased blood ammonia level

(D) Elevated alkaline phosphatase level The findings suggest obstructive jaundice from biliary tract disease (e.g., gallstones). Elevation of the serum alkaline phosphatase level is characteristic of cholestasis. The alkaline phosphatase comes from bile duct epithelium and hepatocyte canalicular membrane.

43 A 51-year-old woman has been feeling increasingly tired for the past 7 months. There are no remarkable findings on physical examination. Laboratory studies include hemoglobin, 9.5 g/dL; hematocrit, 29.1%; *MCV, 124 µm3*; platelet count, 268,000/mm3; and WBC count, 8350/mm3. The reticulocyte index is low. *Hypersegmented polymorphonuclear leukocytes are found* on a peripheral blood smear. Antibodies to which of the following are most likely to be found in this patient? □ (A) Gliadin □ (B) Tropheryma whippelii □ (C) Helicobacter pylori □ (D) Gastric H+ ,K+-ATPase □ (E) Intrinsic factor receptor

(D) Gastric H+ ,K+-ATPase This patient has a megaloblastic anemia with a high MCV. Delayed maturation of the myeloid cells leads to hypersegmentation of polymorphonuclear leukocytes. She most likely has pernicious anemia, resulting from autoimmune atrophic gastritis. Loss of gastric parietal cells from autoimmune injury causes a deficiency of intrinsic factor. In the absence of this factor, vitamin B12 cannot be absorbed in the distal ileum. Among the various "antiparietal cell" antibodies are those directed against the acid-producing "proton pump" enzyme H+,K+-ATPase.

37 A 20-year-old woman in her ninth month of pregnancy has increasing pain on defecation and notices bright red blood on the toilet paper. She has had no previous gastrointestinal problems. After she gives birth, the rectal pain subsides, and there is no more bleeding. Which of the following is the most likely cause of these findings? □ (A) Angiodysplasia □ (B) Ischemic colitis □ (C) Intussusception □ (D) Hemorrhoids □ (E) Volvulus

(D) Hemorrhoids This patient has hemorrhoids. This is a common problem that can stem from any condition that increases venous pressure and causes dilation of internal or external hemorrhoidal veins above and below the anorectal junction.

57 A 45-year-old woman has had increasing abdominal distention for the past 6 weeks. On physical examination, there is an abdominal fluid wave, and bowel sounds are present. Paracentesis yields 1000 mL of slightly cloudy serous fluid. Cytologic examination of the fluid shows malignant cells consistent with adenocarcinoma. The patient's medical history indicates that she has had no major medical illnesses and no surgical procedures. *Which of the following conditions is most likely to have preceded the development of the adenocarcinoma?* □ (A) Angiodysplasia □ (B) Crohn disease □ (C) Diverticulosis □ (D) Hereditary nonpolyposis colon carcinoma □ (E) Hirschsprung disease □ (F) Peptic ulcer disease

(D) Hereditary nonpolyposis colon carcinoma Of the conditions listed, the one most likely to lead to adenocarcinoma in a patient of this age is hereditary nonpolyposis colorectal carcinoma.

30 A 41-year-old man has been HIV positive for the past 8 years and has been receiving highly active antiretroviral therapy for the past year. For the past 2 weeks, he has experienced pain when swallowing. He has had no episodes of hematemesis and no nausea or vomiting. There are no remarkable findings on physical examination. The CD4+ lymphocyte count is now 285/µL. Which of the following conditions is most likely to produce these findings? □ (A) Esophageal squamous cell carcinoma □ (B) Achalasia □ (C) Lower esophageal fibrosis with stenosis □ (D) Herpes simplex esophagitis □ (E) Gastroesophageal reflux disease

(D) Herpes simplex esophagitis A patient who is HIV-positive and has low CD4+ cell counts is at great risk of developing infections. Herpes simplex and Candida are the most likely upper gastrointestinal infections involving the esophagus.

1 Three weeks after a meal at the Trucker's Cafe, a 28-year-old man develops malaise, fatigue, and loss of appetite. He notes passing dark urine. On physical examination, he has mild scleral icterus and right upper quadrant tenderness. Laboratory studies show serum AST of 62 U/L and ALT of 58 U/L. The total bilirubin concentration is 3.9 mg/dL, and the direct bilirubin concentration is 2.8 mg/dL. His symptoms abate over the next 3 weeks. On returning to the cafe, he finds that it has been closed by the city's health department. Which of the following serologic test results is most likely to be positive in this patient? □ (A) Anti-HBs □ (B) IgM anti-HDV □ (C) Anti-HCV □ (D) IgM anti-HAV □ (E) Anti-HBc

(D) IgM anti-HAV This man developed a mild, self-limited liver disease after a meal at a restaurant. He most likely developed HAV infection by consumption of contaminated food or water. The presence of IgM anti-HAV indicates recent infection. The IgM antibody is replaced within a few months by IgG antibodies, which give lifelong immunity to reinfection. The incubation period for HAV infection is short, and the illness is short and mild, with no significant tendency to develop chronic hepatitis. The most common mode of infection for HAV is via the fecal-oral route.

19 A 52-year-old man has had a 6-kg weight loss and nausea for the past 6 months. He has no vomiting or diarrhea. On physical examination, there are no remarkable findings. *Upper gastrointestinal endoscopy shows a 6-cm area of irregular, pale fundic mucosa and loss of the rugal folds*. A biopsy specimen shows a monomorphous infiltrate of lymphoid cells. *Helicobacter pylori* organisms are identified in mucus overlying adjacent mucosa. The patient *receives antibiotic therapy for H. pylori, and the repeat biopsy specimen shows a resolution of the infiltrate*. What is the most likely diagnosis? □ (A) Chronic gastritis □ (B) Diffuse large B-cell lymphoma □ (C) Autoimmune gastritis □ (D) Mucosa-associated lymphoid tissue tumor □ (E) Crohn disease □ (F) Gastrointestinal stromal tumor

(D) Mucosa-associated lymphoid tissue tumor Certain gastrointestinal lymphomas that arise from mucosa-associated lymphoid tissue (MALT) are called MALT lymphomas. Gastric lymphomas that occur in association with Helicobacter pylori infection are composed of monoclonal B cells, whose growth and proliferation depend on cytokines derived from T cells that are sensitized to H. pylori antigens. Treatment with antibiotics eliminates H. pylori and the stimulus for B-cell growth. MALT lesions can occur anywhere in the gastrointestinal tract, although they are rare in the esophagus and appendix. In H. pylori chronic gastritis, which may precede lymphoma development, there are lymphoplasmacytic mucosal infiltrates. Diffuse large B-cell lymphomas and other non-Hodgkin lymphomas that are not MALT lymphomas do not regress with antibiotic therapy.

35 A 22-year-old woman has had several episodes of aspiration of food associated with difficulty swallowing during the past year. On auscultation, crackles are heard at the base of the right lung. A barium swallow shows marked esophageal dilation above the level of the lower esophageal sphincter. A biopsy specimen from the lower esophagus shows an absence of the myenteric ganglia. What is the most likely diagnosis? □ (A) Hiatal hernia □ (B) Plummer-Vinson syndrome □ (C) Barrett esophagus □ (D) Systemic sclerosis □ (E) Achalasia

(E) Achalasia In achalasia, there is incomplete relaxation of the lower esophageal sphincter. Most cases are "primary" or of unknown origin. They are believed to be caused by degenerative changes in neural innervation; the myenteric ganglia are usually absent from the body of the esophagus. There is a long-term risk of development of squamous cell carcinoma.

34 A 56-year-old man has had increasing abdominal enlargement for 6 months. During the past 2 days, he developed a high fever. On physical examination, his temperature is 38.5°C. The abdomen is enlarged and diffusely tender, and there is a fluid wave. Paracentesis yields 500 mL of cloudy yellowish fluid. The cell count is 532/µL with 98% neutrophils and 2% mononuclear cells. A blood culture is positive for Escherichia coli. The patient dies. Which of the following underlying diseases most commonly accounts for these findings? □ (A) α1-Antitrypsin deficiency □ (B) HEV infection □ (C) Hereditary hemochromatosis □ (D) Primary sclerosing cholangitis □ (E) Chronic alcoholism

(E) Chronic alcoholism The diffuse nodularity with depressed scars between the nodules is characteristic of cirrhosis, which led to his ascites complicated by spontaneous bacterial peritonitis and septicemia. The most common cause of cirrhosis in the Western world is alcohol abuse.

13 A 58-year-old woman has experienced gradually increasing malaise, icterus, and loss of appetite for the past 6 months. On physical examination, she has generalized jaundice and scleral icterus. She has mild right upper quadrant tenderness; the liver span is normal. Laboratory studies show total serum bilirubin of 7.8 mg/dL, AST of 190 U/L, ALT of 220 U/L, and alkaline phosphatase of 26 U/L. A liver biopsy is done, and microscopic examination of the biopsy specimen shows piecemeal necrosis of hepatocytes at the limiting plate, with portal bridging fibrosis and a mononuclear infiltrate in the portal tracts. These findings are most typical of which of the following conditions? □ (A) HAV infection □ (B) Congestive heart failure □ (C) Choledocholithiasis □ (D) Hemochromatosis □ (E) HCV infection □ (F) Metastatic breast carcinoma □ (G) Sclerosing cholangitis

(E) HCV infection This patient has clinical evidence of liver disease that has persisted for 6 months, and histologic evidence of hepatic necrosis with portal inflammation and fibrosis. These are features of chronic hepatitis. Of all the hepatitis viruses, HCV is most likely to produce chronic hepatitis, and HAV is the least likely to produce chronic disease. Hepatic congestion with right-sided heart failure produces centrilobular necrosis, but not portal fibrosis.

12 Over the past 4 days, a previously healthy, 38-year-old woman has become increasingly obtunded. On physical examination, she has scleral icterus. She is afebrile, and her blood pressure is 110/55 mm Hg. Laboratory findings show a prothrombin time of 38 seconds, serum ALT of 1854 U/L, AST of 1621 U/L, albumin of 1.8 g/dL, and total protein of 4.8 g/dL. Which of the following additional serologic test results is most likely to be reported in this patient? □ (A) Increased alkaline phosphatase level □ (B) Anti-HCV □ (C) Increased amylase level □ (D) Positive ANA □ (E) Increased ammonia level

(E) Increased ammonia level The patient's history points to an acute fulminant hepatitis with massive hepatic necrosis. The loss of hepatic function from destruction of 80% to 90% of the liver results in hyperammonemia from the defective hepatocyte urea cycle.

34 An 11-month-old, previously healthy infant has not produced a stool for 1 day. The mother notices that the infant's abdomen is distended. On physical examination, the infant's abdomen is very tender, and bowel sounds are nearly absent. An abdominal plain film radiograph shows *no free air, but there are distended loops of small bowel with air-fluid levels*. Which of the following is most likely to produce these findings? □ (A) Meckel diverticulum □ (B) Duodenal atresia □ (C) Hirschsprung disease □ (D) Pyloric stenosis □ (E) Intussusception

(E) Intussusception The infant has signs and symptoms of acute bowel obstruction. Intussusception occurs when one small segment of small bowel becomes telescoped into the immediately distal segment. This disorder can have sudden onset in infants and may occur in the absence of any anatomic abnormality.

53 A 68-year-old woman with a history of rheumatic heart disease is hospitalized with severe congestive heart failure. Several days after admission, she develops abdominal distention. On physical examination, she is afebrile. The abdomen is tympanitic, without a fluid wave, and bowel sounds are absent. A stool sample is positive for occult blood. An abdominal plain film shows no free air. Colonoscopy shows *patchy areas of mucosal erythema with some overlying tan exudate in the ascending and descending colon*. No polyps or masses are found. What is the most likely diagnosis? □ (A) Ulcerative colitis □ (B) Volvulus □ (C) Shigellosis □ (D) Mesenteric vasculitis □ (E) Ischemic colitis

(E) Ischemic colitis Hypotension with hypoperfusion from heart failure is a common cause of ischemic bowel in hospitalized patients. The ischemic changes begin in scattered areas of the mucosa and become confluent and transmural over time. This can give rise to paralytic ileus and bleeding from the affected regions of the bowel mucosa.

48 A 38-year-old man who has been HIV positive for 10 years has had severe nausea and vomiting for the past 2 weeks. On physical examination, he is afebrile. A stool sample is positive for occult blood. The abdomen is not distended, there are no palpable masses or organomegaly, and bowel sounds are present. The patient has oral thrush. There are several reddish purple, 0.5- to 1-cm nodules on the skin of the trunk. Laboratory studies show a CD4+ lymphocyte count of 118/µL. Upper gastrointestinal endoscopy shows 12 reddish purple, 0.6- to 1.8-cm, gastric mucosal nodules. A biopsy specimen of the nodules is most likely to show which of the following neoplasms? □ (A) Adenocarcinoma □ (B) Non-Hodgkin lymphoma □ (C) Carcinoid tumor □ (D) Gastrointestinal stromal tumor □ (E) Kaposi sarcoma □ (F) Peutz-Jeghers polyp □ (G) Squamous cell carcinoma □ (H) Tubular adenoma

(E) Kaposi sarcoma Kaposi sarcoma, non-Hodgkin lymphoma, and anorectal squamous carcinoma are the only neoplasms listed that define AIDS in patients with HIV infection. Kaposi sarcoma most often involves the skin, but it can be found anywhere in the body, including the gastrointestinal tract. Kaposi sarcoma is a vascular lesion—hence the color.

45 A 24-year-old man sees his physician because of abdominal pain and increasing fatigue that has developed over the past 6 months. On physical examination, he is afebrile and appears pale. On palpation, there is mild pain in the right lower quadrant of the abdomen. There are no masses, and bowel sounds are active. Laboratory studies show hemoglobin, 8.9 g/dL; hematocrit, 26.7%; MCV, 74 µm3; platelet count, 255,000/mm3; and WBC count, 7780/mm3. Upper gastrointestinal endoscopy and colonoscopy showed no lesions. One month later, the patient continues to experience the same abdominal pain. Which of the following is most likely to cause this patient's illness? □ (A) Acute appendicitis □ (B) Angiodysplasia □ (C) Celiac sprue □ (D) Diverticulosis □ (E) Meckel diverticulum

(E) Meckel diverticulum

26 A 44-year-old woman has noticed increasingly severe generalized pruritus for the past 8 months. Serum levels of alkaline phosphatase and cholesterol are elevated; antimitochondrial antibody titer is elevated, but ANAs are not present. The serum total bilirubin concentration increases. A liver biopsy is done, and the biopsy specimen shows nonsuppurative, granulomatous destruction of medium-sized bile ducts. Which of the following conditions is most likely to be present? □ (A) α1-Antitrypsin deficiency □ (B) Autoimmune hepatitis □ (C) Choledocholithiasis □ (D) Hereditary hemochromatosis □ (E) Primary biliary cirrhosis □ (F) Primary sclerosing cholangitis □ (G) Wilson disease

(E) Primary biliary cirrhosis The presence of obstructive jaundice, granulomatous destruction of bile ducts, and elevated titers of antimitochondrial antibodies is characteristic of primary biliary cirrhosis. This is an autoimmune condition that may be associated with other autoimmune phenomena (e.g., scleroderma, thyroiditis, glomerulonephritis). Cirrhosis is a late complication of

31 A 67-year-old woman has experienced severe nausea, vomiting, early satiety, and a 9-kg weight loss over the past 4 months. On physical examination, she has mild muscle wasting. Upper gastrointestinal endoscopy shows that the entire gastric mucosa is eroded and has an erythematous, cobblestone appearance. Upper gastrointestinal radiographs show that the stomach is small and shrunken. Which of the following is most likely to be found on histologic examination of a gastric biopsy specimen? □ (A) Early gastric carcinoma □ (B) Gastrointestinal stromal tumor □ (C) Granulomatous inflammation □ (D) Chronic atrophic gastritis □ (E) Signet-ring cell adenocarcinoma

(E) Signet-ring cell adenocarcinoma The endoscopy findings describe the linitis plastica ("leather bottle") appearance of diffuse gastric carcinoma. Histologically, these carcinomas are composed of gastric-type mucus cells that infiltrate the stomach wall diffusely. The individual tumor cells have a signet-ring appearance because the cytoplasmic mucin pushes the nucleus to one side.

18 A 30-year-old man has sudden onset of hematemesis after a weekend in which he consumed large amounts of alcohol. The bleeding stops, but he has another episode under similar circumstances 1 month later. Upper gastroesophageal endoscopy shows longitudinal tears at the esophagogastric junction. What is the most likely mechanism to cause his hematemesis? □ (A) Absent myenteric ganglia □ (B) Autoimmune inflammation □ (C) Herpes simplex virus infection □ (D) Portal hypertension □ (E) Vomiting □ (F) Widened diaphragmatic crura

(E) Vomiting This man has Mallory-Weiss syndrome with esophageal tears from severe vomiting. Most cases occur in the context of alcohol abuse. The bleeding is usually not as life-threatening as varices.

27 A 36-year-old woman is in the sixth month of her first pregnancy, but she is unsure of her dates because she was taking oral contraceptives at the time she became pregnant. She experiences sudden onset of severe abdominal pain. On physical examination, she is afebrile and normotensive. There is right upper quadrant tenderness on palpation. An ultrasound scan of the abdomen shows a well-circumscribed, 7-cm subcapsular hepatic mass. Paracentesis yields bloody fluid. At laparotomy, the mass in the right lower lobe, which has ruptured through the liver capsule, is removed. The remaining liver parenchyma appears to be of uniform consistency, and the liver capsule is otherwise smooth. Which of the following lesions is the most likely diagnosis? □ (A) Adenocarcinoma of the gallbladder □ (B) Cholangiocarcinoma □ (C) Choledochal cyst □ (D) Choriocarcinoma □ (E) Focal nodular hyperplasia □ (F) Hepatic adenoma □ (G) Hepatoblastoma □ (H) Hepatocellular carcinoma

(F) Hepatic adenoma This patient has a circumscribed mass in the liver, suggesting a benign tumor, such as hepatic adenoma. These tumors, which may develop in young women who have used oral contraceptives, can enlarge and rupture from estrogenic stimulation during pregnancy.

49 After a heavy bout of drinking over the weekend, a 38-year-old man feels acutely ill with nausea, upper abdominal pain, and jaundice. On physical examination, there is right upper quadrant tenderness. Laboratory studies include a total WBC count of 16,120/mm3 with 82% segmented neutrophils, 8% bands, 8% lymphocytes, and 2% monocytes. The total serum bilirubin is 4.9 mg/dL, AST is 542 U/L, ALT is 550 U/L, and alkaline phosphatase is 118 U/L. A liver biopsy is done, and it shows *globular eosinophilic cytoplasmic inclusions*. Which of the following terms best describes these findings? □ (A) Apoptosis □ (B) Centrilobular congestion □ (C) Cholestasis □ (D) Hemosiderin deposition □ (E) Hepatocellular carcinoma □ (F) Mallory bodies □ (G) Periportal PAS-positive globules

(F) Mallory bodies This is a classic case of acute alcoholic hepatitis. The figure shows globular eosinophilic cytoplasmic inclusions called Mallory bodies. These inclusions are characteristic of, but not specific for, alcoholic hepatitis. There also are areas of hepatocyte necrosis surrounded by neutrophils. Some neutrophils can be seen in the figure. Apoptosis does not produce widespread necrosis because single cells are involved; this is most typical of viral hepatitis. Centrilobular congestion can lead to centrilobular necrosis without inflammation or Mallory bodies. Cholestasis is marked by plugs of yellow-green bile in canaliculi. Hemosiderin appears granular and brown on H&E staining, but it is blue with Prussian blue stain. Hepatocellular carcinoma can arise in the setting of chronic alcoholism with cirrhosis, but the architecture is distorted, and the cells appear atypical. Periportal PAS-positive globules are characteristic of α1-antitrypsin deficiency; the globules tend to be smaller than Mallory bodies, and there is no acute inflammation. BP7 612-614BP8 650PBD7 904-905PBD8 858

48 A 27-year-old man with a history of intravenous drug use has been known to be infected with hepatitis B virus for the past 6 years and not ill. He is seen in the emergency department because he has had nausea, vomiting, and passage of dark-colored urine for the past week. Physical examination shows scleral icterus and mild jaundice. There are recent and healed track marks in the right antecubital fossa. Neurologic examination shows a confused, somnolent man oriented only to person. He exhibits asterixis. Laboratory studies show total protein, 5 g/dL; albumin, 2.7 g/dL; AST, 2342 U/L; ALT, 2150 U/L; alkaline phosphatase, 233 U/L; total bilirubin, 8.3 mg/dL; and direct bilirubin, 4.5 mg/dL. What is the most likely diagnosis? □ (A) Acute HAV infection □ (B) Acute HBV infection □ (C) Acute HCV infection □ (D) Chronic HAV infection □ (E) Chronic HBV infection □ (F) Chronic HCV infection □ (G) Coinfection with HBV and HDV □ (H) Superinfection of chronic HBV with HDV

(H) Superinfection of chronic HBV with HDV This patient has evidence of superinfection of HDV on chronic hepatitis caused by HBV. The evidence for chronic hepatitis B is the presence of HBsAg and anti-HBc IgG antibody in the absence of anti-HBc IgM antibody. Confirmatory serologic evidence of recent HDV infection is the presence of anti-HDV IgM antibodies. HBV and HDV infections are likely to occur in drug users who inject parenterally. HDV cannot replicate in the absence of HBV; isolated HDV infection does not occur. When HDV infection is superimposed on chronic HBV, three outcomes are possible: mild hepatitis B may be converted to fulminant disease; acute hepatitis may occur in an asymptomatic HBV carrier; or chronic progressive disease may develop, culminating in cirrhosis. BP7 604-605BP8 644PBD7 891, 895-896PBD8 848-849

Question 5 A 38-year-old man has had upper abdominal pain for 3 months. For the past week he has had nausea. On physical examination a stool sample is positive for occult blood. An upper GI endoscopy reveals no esophageal lesions, but there is a solitary 2 cm diameter shallow, sharply demarcated ulceration of the stomach. Which of the following is most characteristic for this lesion? A Antral location B Potential for metastases C Increased gastric acid production. D No need for biopsy E Accompanying pancreatic gastrinoma

A Antral location The gastric antrum is the typical location for a benign peptic ulcer. The small size and sharp margins are characteristics for a benign ulcer, but all gastric ulcerations require biopsy because malignancies have many different appearances.

Question 25 A 53-year-old postmenopausal woman has been feeling increasingly tired for 3 months. She has had mid-epigastric pain for the past month. On physical examination her stool is *positive for occult blood*. Laboratory findings include a WBC count of 6300/uL, Hgb 12.0 g/dL, Hct 35.4%, MCV 73 fL, platelet count 236,000/uL, total protein 6.0 g/dL, albumin 3.5 g/dL, *elevated alkaline phosphatase*, AST 84 U/L, ALT 35 U/L, lipase 45 U/L, *total bilirubin 1.1 mg/dL*, and direct bilirubin 0.8 mg/dL. Which of the following is the most likely explanation for her findings? A Metastatic adenocarcinoma B Wilson disease C Acute hepatitis A D Acetaminophen toxicity E Congestive heart failure F Chronic alcohol abuse G Non-alcoholic fatty liver

A Metastatic adenocarcinoma The elevation in the alkaline phosphatase in the absence of an elevated bilirubin suggests focal hepatic obstruction, which could occur with metastases. The positive stool guaiac suggests a colonic primary.

Question 13 A 72-year-old woman notes increasing *jaundice* and nausea for the past month. On physical examination she is *afebrile, but scleral icterus is present. There is no abdominal pain on palpation*. She has active bowel sounds. A stool sample is negative for occult blood. Laboratory findings include total protein 6.1 g/dL, albumin 3.3 g/dL, alkaline phosphatase 210 U/L, AST 49 U/L, ALT 40 U/L, *total bilirubin 7.2 mg/dL, and direct bilirubin 6.3 mg/dL*. Her serum lipase is 50 U/L. Which of the following conditions is she most likely to have? A Pancreatic adenocarcinoma B Cystic fibrosis C Chronic active hepatitis D Primary biliary cirrhosis E Chronic persistent hepatitis F Autoimmune hemolytic anemia

A Pancreatic adenocarcinoma Adenocarcinoma of the head of pancreas produces extrahepatic biliary obstruction with an elevation predominantly of the direct bilirubin along with an elevation in alkaline phosphatase. This results in the classic finding of 'painless jaundice'. There is no significant inflammatory component.

Question 6 A 15-year-old boy from Ghana has the acute onset of right upper quadrant abdominal pain. Abdominal ultrasound reveals a dilated gallbladder with thickened wall and filled with calculi. A laparoscopic cholecystectomy is performed. The gallbladder is opened to reveal ten multifaceted 0.5 to 1 cm diameter dark, greenish-black gallstones. Which of the following underlying conditions does this boy most likely have? A Sickle cell anemia B Crohn disease C Hypercholesterolemia D Hyperparathyroidism E Primary biliary cirrhosis

A Sickle cell anemia Darkly pigmented gallstones usually contain bilirubin. Hyperbilirubinemia is a consequence of hemolysis. Patients with sickle cell anemia have chronic hemolysis.

Question 18 A 41-year-old man has experienced progressive fatigue, pruritus, and icterus for the past 4 months. He had a total colectomy performed five years ago. On physical examination he is afebrile. Scleral icterus is present. His stool from a Koch pouch is negative for occult blood. Cholangiography reveals the widespread obliteration of intrahepatic bile ducts. A liver biopsy is performed and on microscopic examination shows periductular 'onion skin' fibrosis with a moderate lymphocytic infiltrate. Some intrahepatic bile ducts are obliterated, but there is no interface hepatitis. Which of the following underlying diseases is he most likely to have? A Ulcerative colitis B Systemic lupus erythematosus C Wilson disease D Hepatitis B viral infection E Primary biliary cirrhosis F Alpha-1-antitrypsin deficiency

A Ulcerative colitis Sclerosing cholangitis is one of the extra-intestinal manifestations of inflammatory bowel disease and is seen most frequently with ulcerative colitis. The colectomy was probably performed because of his increasing risk for adenocarcinoma, but even after colectomy extraintestinal manifestations of UC can occur.

Question 8 A 63-year-old woman has increasing fatigue for the past year. On physical examination no abnormal findings are noted. Laboratory studies show microcytic hypochromic anemia. Upper GI endoscopy and colonoscopy are performed with no lesions noted. Capsule endoscopy reveals a 2 cm diameter slightly raised dark red mucosal area with with scalloped edges and a visible draining vein. What is the most likely diagnosis? A Adenocarcinoma B Angiodysplasia C Celiac disease D Ectopic gastric tissue E Ischemic bowel disease

B Angiodysplasia Angiodysplasia is the most common vascular lesion of the gastrointestinal tract. It can be asymptomatic or produce gastrointestinal bleeding. It is more prevalent after age 50. Bleeding may be intermittent. Angiodysplastic lesions have ectatic nests of tortuous veins, venules, and capillaries. The vascular channels may be separated from the intestinal lumen by the thin vascular wall and attenuated epithelium.Chronic venous obstruction may play a role. The cecum has the largest diameter of any colonic segment with the most wall tension, explaining why this is the most common site.

Question 4 A 31-year-old man with a stab wound to the abdomen is taken to surgery. While repairing the small intestine, the surgeon notices the presence of a 1 cm circumscribed submucosal mass in the ileum. The lesion is resected and on gross examination has a firm, yellow-tan cut surface. Microscopically, the mass is composed of nests of cells with uniform small round nuclei and cytoplasm with small purple granules. The cytoplasm is positive with antibody to chromogranin on immunohistochemical staining. Which of the following pathologic findings is most likely to accompany this man's lesion? A Liver metastases B Another similar lesion C Multiple gastric ulcerations D Pancreatic adenocarcinoma E Inflammatory bowel disease F Tropheryma whippelii infection

B Another similar lesion This lesion is a carcinoid tumor, and these neoplasms can be multiple, even when biologically benign. Small carcinoids typically act in a benign fashion. Most of them do not secrete hormones causing clinical symptoms, but some do. They are often incidental findings.

Question 31 A 54-year-old man has complained for 5 months of upper abdominal pain accompanied by nausea. He does not have hematemesis. On physical examination the only finding is a stool sample positive for occult blood. Upper GI endoscopy is performed and gastric biopsies are taken that on microscopic examination reveal *acute and chronic mucosal inflammation along with the presence of Helicobacter pylori organisms*. The presence of these organisms is most likely to be associated with which of the following? A Gastric mucosal invasion with septicemia B Duodenal peptic ulceration C Pernicious anemia D Hypochlorhydria E Diffuse large B cell lymphoma

B Duodenal peptic ulceration H. pylori is present in about 90% of cases of chronic gastritis and nearly 100% of cases of duodenal ulceration. H. pylori is present with increasing frequency with age, which suggests that it is an important cause for chronic gastritis in the elderly.

A 41-year-old man has a history of drinking 1 to 2 liters of whisky per day for the past 20 years. He has had numerous episodes of nausea and vomiting in the past 5 years. He now experiences a bout of prolonged vomiting, followed by massive hematemesis. On physical examination his vital signs are: T 36.9°C, P 110/min, RR 26/min, and BP 80/40 mm Hg lying down. His heart has a regular rate and rhythm with no murmurs and his lungs are clear to auscultation. There is no abdominal tenderness or distension and bowel sounds are present. His stool is negative for occult blood. Which of the following is the most likely diagnosis? A Hiatal hernia B Esophageal laceration C Esophageal pulsion diverticulum D Barrett esophagus E Esophageal squamous cell carcinoma F Esophageal stricture

B Esophageal laceration This is Mallory-Weiss syndrome. The lacerations are induced by the forceful, prolonged vomiting and can extend to submucosal veins that bleed profusely. Esophageal variceal bleeding should also be suspected with such a history, because hepatic cirrhosis is likely to be present. The acute nature of the process means blood has not yet passed through the bowel to the rectum

Question 26 A 25-year-old man complains of a low volume but chronic, foul smelling diarrhea for the past year. He has no nausea or vomiting. On physical examination there is no abdominal pain or masses and bowel sounds are present. His stool is negative for occult blood. Laboratory studies include a quantitative stool fat of 10 g/day. Upper GI endoscopy is performed with biopsies taken of the duodenum, and on microscopic examination show absence of villi, increased surface intraepithelial lymphocytes, and hyperplastic appearing crypts. Which of the following therapies is most likely to be useful for this man? A Antibiotics B Gluten-free diet C Selective vagotomy D Corticosteroids E Segmental duodenal resection F Aromatherapy

B Gluten-free diet He has celiac disease from ingestion of grains (wheat, rye, barley, oats) that contain gluten with gliaden protein. The enzyme tissue transglutamidase breaks down the gliaden into peptides which, when displayed to antigen presenting cells, activate CD4 lymphocytes that produce the mucosal inflammation.

Question 2 A 50-year-old man has had persistent nausea for 5 years with occasional vomiting. On physical examination there are no abnormal findings. He undergoes upper GI endoscopy, and a *small area of gastric fundal mucosa has loss of rugal folds*. Biopsies are taken and microscopically reveal *well-differentiated adenocarcinoma confined to the mucosa*. An upper GI endoscopy performed 5 years previously showed a pattern of gastritis and microscopically there was chronic inflammation with the presence of. Which of the following is the most likely risk factor for his neoplasm? A Inherited APC gene mutation B Helicobacter pylori infection C Chronic alcohol abuse D Use of non-steroidal anti-inflammatory drugs E Vitamin B12 deficiency

B Helicobacter pylori infection H. pylori infection can lead to chronic gastritis that promotes development of adenocarcinoma. The prognosis with early gastric carcinoma (EGC) is good, compared with other gastric cancers.

Question 35 A 38-year-old woman has had episodes of light-headedness, irritability, and difficulty concentrating at her work for the past year. On physical examination there are no abnormal findings. Laboratory studies show her hemoglobin is 14.2 g/dL. An abdominal CT scan shows a 1 cm mass in the tail of the pancreas. Which of the following is the most likely diagnosis? A Cystadenoma B Insulinoma C Adenocarcinoma D Pseudocyst E Pheochromocytoma F Neuroendocrine carcinoma

B Insulinoma A variety of hormones can potentially be elaborated by an islet cell adenoma, or pancreatic neuroendocrine tumor, but the most common is insulin, leading to hypoglycemia. This hypoglycemia may occur periodically. Often, the adenoma is so small that it is difficult to detect by radiologic imaging techniques.

Question 16 A 44-year-old man, an emergency medical technician, has felt fatigued for the past 4 months. He experienced an episode of jaundice 10 years ago, but that resolved and he has been healthy since. On physical examination there are no remarkable findings. Laboratory studies show his hemoglobin is 14 g/dL and serum electrolytes normal, but he has a total protein of 5.4 g/dL, albumin 2.9 g/dL, ALT 132 U/L and AST 113 U/L with total bilirubin 1.3 mg/dL and direct bilirubin 0.8 mg/dL. A liver biopsy is performed and microscopic examination shows interface inflammation with extension of inflammation into the lobules from the triads. There is focal ballooning degeneration of hepatocytes. Which of the following laboratory test findings is most characteristic for his disease? A Decreased serum alpha-1-antitrypsin B Positive hepatitis B surface antigen C Increased serum ferritin D Decreased serum ceruloplasmin E Positive antimitochondrial antibody

B Positive hepatitis B surface antigen This is chronic active hepatitis with hepatitis B infection. This illustrates the wisdom of getting the series of hepatitis B vaccinations. Though most persons clear the virus, a significant number go on to chronic liver disease.

Question 24 A 45-year-old woman has noted difficulty swallowing for the past 6 months. On physical examination there are no abnormal findings. A barium swallow reveals an area of stricture in the lower esophagus just above the gastroesophageal junction. She has an upper GI endoscopy performed and biopsies of the lower esophagus are taken which show squamous epithelium with no acute or chronic inflammation or ulceration, only submucosal atrophy along with fibrosis of smooth muscle. Which of the following is the most likely diagnosis? A Mallory-Weiss syndrome B Systemic sclerosis C Iron deficiency D Portal hypertension E Barrett esophagus

B Systemic sclerosis Patients with progressive systemic sclerosis with either the limited form of the disease (sometimes called CREST syndrome) or diffuse scleroderma have fibrosis in the gastrointestinal tract, most often involving the lower esophagus. An endoscopic procedure to dilate the esophagus may need to be performed periodically to improve swallowing.

Question 32 A 46-year-old man has had *malabsorption* with 5 kg weight loss for the past year associated with a low volume diarrhea. He also has a *polyarthritis and complains of occasional visual hallucinations*. On physical examination there are no joint deformities. A stool sample is negative for occult blood. An abdominal CT scan reveals no masses, only generalized lymphadenopathy. On upper GI endoscopy, there are no esophageal or gastric lesions, but there are *broad, flattened villi in the duodenum*. Biopsies of the duodenum reveal *numerous PAS-positive macrophages in the submucosa*. Which of the following therapies is most likely to be useful for this man? A Gluten-free diet B Corticosteroids C Antibiotics D Segmental duodenal resection E Antacids F Aromatherapy

C Antibiotics Whipple disease is rare. It can affect multiple organs, including the brain. The organisms causing Whipple disease are actinomycetes (Tropheryma whippelii). The foamy macrophages can be in the small intestinal submucosa, adjacent lymph nodes, or at extraintestinal sites.

Question 3 A 58-year-old man has had increasing difficulty swallowing for the past 6 months and has lost 5 kg. No abnormal physical examination findings are noted. Upper GI endoscopy reveals a nearly circumferential mass with overlying ulceration in the mid esophageal region. Biopsy of the mass reveals pink polygonal cells with marked hyperchromatism and pleomorphism. Which of the following is the most likely risk factor for development of his disease? A Iron deficiency B Helicobacter pylori infection C Chronic alcohol abuse D High fruit diet E Zenker diverticulum

C Chronic alcohol abuse Chronic alcoholism and tobacco use are two of the most important risk factors for squamous cell carcinoma of the esophagus in the U.S., specifically related to squamous cell carcinomas of the mid-esophagus, as in this man. Zinc and molybdenum are trace elements in the diet whose absence increases the risk for carcinoma of the esophagus. Food contaminated with Aspergillus also carries a risk, as does food containing nitrosamines.

Question 30 A 39-year-old woman has experienced *substernal burning pain following meals for the past 15 years*. On physical examination there are no abnormal findings. Upper GI endoscopy is performed and there are 1 to 3 cm long tongues of erythematous mucosa extending from the gastroesophageal junction at the Z line upward into the lower esophagus. Biopsies are performed of this region and microscopic examination shows areas of *gastric cardiac-type mucosa and intestinalized mucosa*. Which of the following is the most likely explanation for this woman's findings? A congenital anomaly B Adenocarcinoma C Gastroesophageal reflux D Diverticulum formation E Iron deficiency anemia F Systemic sclerosis G Alcohol abuse

C Gastroesophageal reflux There is columnar metaplasia in the lower esophagus with Barrett esophagus from gastroesophageal reflux disease. A small percentage of these patients may develop adenocarcinoma. In this case there is no dysplasia that presages evolution to carcinoma.

Question 9 A 45-year-old man has had vague abdominal pain and nausea for the past 3 years. This pain is unrelieved by antacid medications. He has no difficulty swallowing and no heartburn following meals. On physical examination there are no abnormal findings. Upper GI endoscopy reveals antral mucosal erythema, but no ulcerations or masses. Biopsies are taken, and microscopically there is a chronic non-specific gastritis. Which of the following conditions is most likely to be present in this man? A Zollinger-Ellison syndrome B Pernicious anemia C Helicobacter pylori infection D Adenocarcinoma E Crohn disease F Mixed connective tissue disease

C Helicobacter pylori infection H. pylori infection is often seen in association with chronic gastritis and with peptic ulcer disease.

Question 12 A 39-year-old man is having a routine physical examination because of a history of colon cancer in his family. He has no abdominal tenderness or masses, and active bowel sounds are present. However, his stool is positive for occult blood. Colonoscopy is performed. There are 7 polyps found in the ascending colon: three of these are small 0.5 cm pedunculated tubular adenomas, three are 1 cm tubulovillous adenomas, and one is a 2 cm sessile villous adenoma in the cecum containing a focus of well differentiated adenocarcinoma. Which of the following is his most likely underlying diagnosis? A Peutz-Jeghers syndrome B Chronic ulcerative colitis C Hereditary non-polyposis colon carcinoma D Adenomatous polyposis coli E Gardner syndrome

C Hereditary non-polyposis colon carcinoma The number of polyps is not great in this case, but a cancer at his age suggests an aggressive course for a polyp. Many polyps and cancers of HNPCC involve the right (ascending) colon. This condition arises from abnormal mismatch repair genes and accounts for perhaps 1 to 3% of all colon cancers.

Question 37 A 35-year-old HIV positive woman known has had pain on swallowing for the past week. No abnormal physical examination findings are noted. Upper GI endoscopy is performed. There are 3 sharply circumscribed 0.3 to 0.8 cm ulcers in the lower esophagus. She is most likely to have infection with which of the following organisms? A Helicobacter pylori B Candida albicans C Herpes simplex virus D Mycobacterium avium-complex E Cytomegalovirus F Human herpes virus 8 G Cryptococcus neoformans

C Herpes simplex virus Herpes esophagitis often has 'punched out' ulcerations. It is seen in immunocompromised patients, including those with HIV/AIDS. Dysphagia with pain is a common presentation. Dissemination is unlikely.

Question 29 A 50-year-old man gives a history of chronic alcohol abuse. He has had bouts of abdominal pain in the past year. For the past month, he has had more frequent and worsening abdominal pain. Physical examination reveals right upper and left upper quadrant pain with guarding. An abdominal plain film radiograph reveals no free air, but there is extensive peritoneal fluid collection along with dilated loops of small bowel. An abdominal CT scan reveals a 7 to 8 cm cystic mass in the tail of the pancreas. Which of the following is the most likely diagnosis? A Pancreatic neuroendocrine tumor B Acute pancreatitis C Pancreatic pseudocyst D Metastatic carcinoma E Pancreatic adenocarcinoma

C Pancreatic pseudocyst A pseudocyst is a complication of chronic pancreatitis seen most frequently in persons with a history of chronic alcoholism. The pseudocyst is an area of necrosis with a wall composed of granulation tissue. Drainage of this pseudocyst is often needed as treatment.

Question 19 A 43-year-old man has had mid epigastric pain and nausea for the past 2 months. On physical examination he has no abnormal findings. On upper GI endoscopy a solitary sharply demarcated 2-cm shallow gastric antral ulcer is seen. Which of the following laboratory test findings is most likely to be present in this man? A Gastric achlorhydria B Positive serology for antinuclear antibody C Positive urea breath test D Increased plasma cortisol E Elevated serum gastrin

C Positive urea breath test Helicobacter pylori organisms in the gastric mucus produce urease which will break down urea to ammonia and to CO2. For this urea breath test the patient drinks a measured quantity of a urea-containing solution with radiolabeled carbon. If the H. pylori organisms are present they metabolize the urea and release the radiolabeled carbon which is detected in exhaled air. Most gastric ulcers of this size are benign. Bleeding is a common complication of ulcer disease, seen in 25 to 33% of cases.

Question 11 A 32-year-old woman has a 10 year history of intermittent, bloody diarrhea. She has no other major medical problems. On physical examination there are no lesions palpable on digital rectal examination, but a stool sample is positive for occult blood. Colonoscopy reveals a friable, erythematous mucosa with focal ulceration that extends from the rectum to the mid-transverse colon. Biopsies are taken and all reveal mucosal acute and chronic inflammation with crypt distortion, occasional crypt abscesses, and superficial mucosal ulceration. This patient is at greatest risk for development of which of the following conditions? A Acute pancreatitis B Diverticulitis C Sclerosing cholangitis D Appendicitis E Perirectal fistula F Non-Hodgkin lymphoma

C Sclerosing cholangitis This patient has ulcerative colitis (UC). One of the extraintestinal manifestations of this form of inflammatory bowel disease is hepatic sclerosing cholangitis, which can still occur even after the colon is removed.

Question 17 A 20-year-old man is healthy but has a family history of colon cancer with onset at a young age. There are no abnormal physical examination findings. He undergoes colonoscopy and there are over 200 tubular adenomas ranging in size from 0.2 to 1 cm on gross inspection and microscopic examination of biopsies. Which of the following genetic diseases is he most likely to have? A Hereditary non-polyposis colon carcinoma syndrome B Gardner syndrome C Peutz-Jeghers syndrome D Adenomatous polyposis coli E Multiple endocrine neoplasia

D Adenomatous polyposis coli Persons with familial polyposis caused by inheritance of an abnormal APC gene should have a total colectomy for this reason. By late childhood to young adulthood, one or more of the polyps will give rise to adenocarcinoma. The polyps do not occur elsewhere in the GI tract.

Question 15 A 62-year-old man has had anorexia, vomiting, and vague abdominal pain accompanied by weight loss of 6 kg over the past 2 months. Physical examination reveals supraclavicular non-tender lymphadenopathy. He becomes progressively cachectic. An abdominal CT scan shows the stomach is shrunken with the gastric wall thickened to 1 cm and with extensive overlying mucosal erosions. Multiple masses from 1 to 4 cm in size are scattered within the liver. Which of the following conditions most likely preceded development of his illness? A Acquired immunodeficiency syndrome B Hyperglycemia C Chronic alcoholism D Pernicious anemia E Systemic sclerosis F Use of NSAIDS

D Pernicious anemia This is linitis plastica of the stomach, which typically has a signet ring cell pattern of adenocarcinoma diffusely infiltrating the stomach. Autoimmune gastritis is a risk factor for this condition. The atrophic gastritis leads to loss of parietal cells that produce intrinsic factor needed for B12 absorption.

Question 20 A 40-year-old man has a long history of chronic alcohol abuse. On physical examination his liver edge is firm on palpation of the abdomen, but liver span does not appear to be increased. An abdominal CT scan reveals a cirrhotic liver. He joins a support group for persons with chronic alcohol abuse and he stops drinking. Despite his continued abstinence from alcohol, he most likely remains at risk for development of which of the following diseases? A Hepatic adenoma B Focal nodular hyperplasia C Cholelithiasis D Angiosarcoma E Hepatocellular carcinoma F Non-Hodgkin lymphoma G Hemangioma

E Hepatocellular carcinoma Hepatocellular carcinoma occurs most often in the setting of cirrhosis (which occurs most often from chronic alcoholism and from chronic viral hepatitis B and C). The liver remodels itself over years, so the cirrhosis is slowly reversible, with decreasing carcinoma risk.

Question 21 A 22-year-old woman has had progressive malaise for the past year. She has become increasingly obtunded over the past week. On physical examination she is afebrile. Laboratory studies show a plasma ammonia of 55 micromol/L along with serum total bilirubin of 5.8 mg/dL, direct bilirubin 4.6 mg/dL, AST 110 U/L, and ALT 135 U/L. Her serum ceruloplasmin is 14 mg/dL. The antimitochondrial antibody test is negative. A liver biopsy is performed and microscopic examination reveals increased copper deposition. Which of the following ocular findings is most likely to be present in this woman? A Bilateral papilledema B Macular degeneration C Proliferative retinopathy D Crystalline lens cataract formation E Corneal Kayser-Fleischer rings F Canal of Schlemm occlusion

E Corneal Kayser-Fleischer rings These pale tan rings can be seen with a slit lamp examination. They are characteristic for Wilson disease, an autosomal recessive disorder with mutation in the ATP7B gene that encodes for a copper-transporting ATPase. Patients have decreased serum ceruloplasmin, the copper-carrying protein, and increased tissue deposition of copper, particularly in liver, eye, and basal ganglia of brain. Untreated cases such as this one can lead to end stage liver disease. Ammonia increases most with acute liver failure.

Question 10 A 25-year-old man has noted cramping abdominal pain for the past week associated with fever and low-volume diarrhea. On physical examination, there is right lower quadrant tenderness. Bowel sounds are present. His stool is positive for occult blood. A colonoscopy reveals mucosal edema and ulceration in the ascending colon, but the transverse and descending portions of the colon are not affected. Laboratory studies show serum anti-Saccharomyces cerevisiae antibodies. Which of the following microscopic findings is most likely to be present in biopsies from his colon? A Crypt abscesses B Entameba histolytica organisms C Adenocarcinoma D Band-like mucosal fibrosis E Non-caseating granulomas F Necrotizing vasculitis

E Non-caseating granulomas This history is most typical for Crohn disease, which is a form of inflammatory bowel disease that tends to involve the bowel in a segmental pattern.

Question 34 A 54-year-old Asian man has had malaise with a 6 kg weight loss over the past 7 months. On physical examination he has a firm, nodular liver edge. His stool is negative for occult blood. Laboratory studies show a *positive serology for hepatitis B surface antigen*, but negative serologies for hepatitis B surface antibody, hepatitis A IgM antibody, and hepatitis C antibody. His serum *alpha-fetoprotein is elevated*. Which of the following neoplasms is he most likely to have? A Hemangioma B Hepatic adenoma C Hepatic angiosarcoma D Bile duct adenocarcinoma E Cholangiocarcinoma F Hepatocellular carcinoma G Non-Hodgkin lymphoma

F Hepatocellular carcinoma Chronic hepatitis and both micro- and macronodular cirrhosis carry an increased risk for liver cancer. Most often this is hepatocellular carcinoma. Either chronic hepatitis B or C infection increases the risk for primary liver cancer. An elevated AFP is characteristic for hepatocellular carcinoma.

Question 23 A 23-year-old primigravida gives birth at term following an uncomplicated pregnancy to a male infant with no apparent congenital anomalies. At 4 weeks of age the infant begins to exhibit forceful vomiting after each feeding. The infant had been fine previously and gaining weight normally. Which of the following conditions is the probable cause for his vomiting? A congenital duodenal atresia B Necrotizing enterocolitis C Mallory-Weiss syndrome D Hirschsprung disease E Tracheo-esophageal fistula F Pyloric stenosis

F Pyloric stenosis The male sex, age, and symptoms are all quite typical for pyloric stenosis, a condition that exhibits the genetic feature called the 'threshold of liability' in which males are more likely to have the disease (lower threshold) if born into a family with girls affected (higher threshold, and thus, more genetic tendencies for the disease to pass on).


Ensembles d'études connexes

chapter 10 core concepts in health 14e

View Set

Personal Finance Chapter 4 - Financial Services: Savings Plans and Payment Accounts

View Set

Exam 1 - Answers and Practice Problems

View Set

Chapter 1: Understanding Social Problems

View Set

Test Questions Insurance Property and Casualty

View Set

Atmospheric Pollution Practice 100%

View Set